Đến nội dung

Hình ảnh

BĐT AM-GM


  • Please log in to reply
Chủ đề này có 339 trả lời

#121
duong vi tuan

duong vi tuan

    Thượng sĩ

  • Thành viên
  • 229 Bài viết
bài 11) ta chứng minh:
$$\prod (a^2+2b^2)\geq \prod (a^2+ab+bc)$$
ta có:
$$(a^2+b^2 +b^2)(a^2+a^2+c^2)\geq (a^2+ab+bc)^2$$
làm tương tự.... từ đó ta có điều phải cm :D

Bài viết đã được chỉnh sửa nội dung bởi duong vi tuan: 29-12-2012 - 11:04

NGU
Hình đã gửi

#122
no matter what

no matter what

    Why not me

  • Thành viên
  • 397 Bài viết
Bài 14,Chứng minh với a,b,c là 3 cạnh 1 tam giác,ta có
$((2a^2+bc)(2b^2+ca)(2c^2+ab)\geq (2a^2+2b^2-c^2)(2b^2+2c^2-a^2)(2c^2+2a^2-b^2)$
Vẫn cách làm như bài 11 cùng các bài kia,xin mời mọi người :lol:
p/s :mọi người hãy xem nỗ lực của chngs ta ở trang này nhé http://diendantoanho...gm/page__st__40
dòng màu đỏ :icon6:

#123
duong vi tuan

duong vi tuan

    Thượng sĩ

  • Thành viên
  • 229 Bài viết

Bài 14,Chứng minh với a,b,c là 3 cạnh 1 tam giác,ta có
$((2a^2+bc)(2b^2+ca)(2c^2+ab)\geq (2a^2+2b^2-c^2)(2b^2+2c^2-a^2)(2c^2+2a^2-b^2)$
Vẫn cách làm như bài 11 cùng các bài kia,xin mời mọi người :lol:
p/s :mọi người hãy xem nỗ lực của chngs ta ở trang này nhé http://diendantoanho...gm/page__st__40
dòng màu đỏ :icon6:

Ủng hộ độ tiến : :D
ta chứng minh: $$(2a^2+bc)^2\geq (2a^2+2b^2-c^2)(2c^2+2a^2-b^2)\Leftrightarrow 2(b-c)^2(a+b+c)(b+c-a)\geq 0$$ (đúng)
tương tự ta có điều phải cm
NGU
Hình đã gửi

#124
no matter what

no matter what

    Why not me

  • Thành viên
  • 397 Bài viết

Ủng hộ độ tiến : :D
ta chứng minh: $$(2a^2+bc)^2\geq (2a^2+2b^2-c^2)(2c^2+2a^2-b^2)\Leftrightarrow 2(b-c)^2(a+b+c)(b+c-a)\geq 0$$ (đúng)
tương tự ta có điều phải cm

Thanks
Bài 15:,Chứng minh với mọi a,b,c dương có tổng bằng 3 ta có BĐT sau
$\frac{a^2+bc}{b+ca}+\frac{b^2+ca}{c+ab}+\frac{c^2+ab}{a+bc}\geq 3$
Bài này mình chẳng biết phải nói thế nào cho dễ hiểu nữa,tuy nó không khó nhưng mà .... rất là chuối :(
nói chung là bằng AM-GM,mọi người sẽ đưa được về chung mẫu :wub:

#125
BoFaKe

BoFaKe

    Thiếu úy

  • Thành viên
  • 613 Bài viết

Thanks
Bài 15:,Chứng minh với mọi a,b,c dương có tổng bằng 3 ta có BĐT sau
$\frac{a^2+bc}{b+ca}+\frac{b^2+ca}{c+ab}+\frac{c^2+ab}{a+bc}\geq 3$
Bài này mình chẳng biết phải nói thế nào cho dễ hiểu nữa,tuy nó không khó nhưng mà .... rất là chuối :(
nói chung là bằng AM-GM,mọi người sẽ đưa được về chung mẫu :wub:

Mình nghĩ là củ chuối thật :wacko:
Theo AM-GM ta có:$\sum \frac{a^{2}+bc}{b+ca}\geq \sum \frac{a^{2}+bc}{\frac{b^{2}+1}{2}+\frac{a^{2}+c^{2}}{2}}= \sum \frac{2(a^{2}+bc)}{a^{2}+b^{2}+c^{2}+1}$
Đến đây không biết đã được chưa,ai giúp mình làm tiếp với :(
-------------------------
P/S:Xì tốp,mọi người thử cách khác đi,cái này nó bị ngược dấu,để mình sửa lại đã :mellow:

Bài viết đã được chỉnh sửa nội dung bởi BoFaKe: 30-12-2012 - 19:52

~~~~~~~~~~~~~~Tiếc gì mà không click vào nút like mọi ngươì nhỉ ^0^~~~~~~~~~~~~~

#126
duong vi tuan

duong vi tuan

    Thượng sĩ

  • Thành viên
  • 229 Bài viết

Thanks
Bài 15:,Chứng minh với mọi a,b,c dương có tổng bằng 3 ta có BĐT sau
$\frac{a^2+bc}{b+ca}+\frac{b^2+ca}{c+ab}+\frac{c^2+ab}{a+bc}\geq 3$
Bài này mình chẳng biết phải nói thế nào cho dễ hiểu nữa,tuy nó không khó nhưng mà .... rất là chuối :(
nói chung là bằng AM-GM,mọi người sẽ đưa được về chung mẫu :wub:

bất đẳng thức trên tương đương:$$\sum \frac{a^2+bc}{(b+a)(b+c)+2ca}\geq 1$$

$$\sum \frac{a^2+bc}{(b+a)(b+c)+2ca}\geq \sum 2\frac{a^2+bc}{(b+c)^2+(b+c)^2+(c+a)^2}\doteq 1$$
NGU
Hình đã gửi

#127
Oral1020

Oral1020

    Thịnh To Tướng

  • Thành viên
  • 1225 Bài viết
Bài 20:
$\text{Solution:}$
$\oplus$Áp dụng bắt đẳng thức $\text{AM-GM}$,ta có:
$b+c \ge 2\sqrt{bc}$
$\oplus$Tương tự,ta có:
$\dfrac{b+c}{\sqrt{a}}+\dfrac{c+a}{\sqrt{b}}+\dfrac{a+b}{\sqrt{c}} \ge 2(\sqrt{\dfrac{bc}{a}}+\sqrt{\dfrac{ca}{b}}+\sqrt{\dfrac{ab}{c}})$
$\oplus$Ta lại có:
$\sqrt{\dfrac{bc}{a}}+\sqrt{\dfrac{ca}{b}}+\sqrt{\dfrac{ab}{c}}=\sum (\sqrt{\dfrac{bc}{a}}+\sqrt{\dfrac{ca}{b}})\ge 2(\sqrt{a}+\sqrt{b}+\sqrt{c} \ge \sqrt{a}+\sqrt{b}+\sqrt{c} +3\sqrt[6]{abc}=\sqrt{a}+\sqrt{b}+\sqrt{c}+3$
Vậy:$\boxed{\dfrac{b+c}{\sqrt{a}}+\dfrac{c+a}{\sqrt{b}}+\dfrac{a+b}{\sqrt{c}} \ge \sqrt{a}+\sqrt{b}+\sqrt{c}+3}$

"If I feel unhappy,I do mathematics to become happy.


If I feel happy,I do mathematics to keep happy."

Alfréd Rényi

Hình đã gửi


#128
no matter what

no matter what

    Why not me

  • Thành viên
  • 397 Bài viết
16,Chứng minh với mọi x,y,z dương ta có
$(\frac{x}{y}+\frac{z}{\sqrt[3]{xyz}})^2+(\frac{y}{z}+\frac{x}{\sqrt[3]{xyz}})^2+(\frac{z}{x}+\frac{y}{\sqrt[3]{xyz}})^2\geq 12$

#129
HungHuynh2508

HungHuynh2508

    Thượng sĩ

  • Thành viên
  • 222 Bài viết
17, Cho x,y,z>0 thõa mãn $\frac{1}{x}+\frac{1}{y}+\frac{1}{z}=4$ . Chứng minh rằng
$\sum \frac{1}{2x+y+z}\leq 1$
Hạnh phúc là cho đi đâu chỉ nhận riêng mình!

7e3c59fbf62d4c5280e6cf2ad53cdcb8.0.gif

#130
banhgaongonngon

banhgaongonngon

    Thượng úy

  • Thành viên
  • 1046 Bài viết

17, Cho x,y,z>0 thõa mãn $\frac{1}{x}+\frac{1}{y}+\frac{1}{z}=4$ . Chứng minh rằng
$\sum \frac{1}{2x+y+z}\leq 1$


$\frac{1}{2x+y+z}\leq \frac{1}{4}\left ( \frac{1}{x+y}+\frac{1}{x+z}\right )\leq \frac{1}{16}\left ( \frac{1}{x} +\frac{1}{x}+\frac{1}{y}+\frac{1}{z}\right )$
Tương tự, ta có đpcm

#131
duong vi tuan

duong vi tuan

    Thượng sĩ

  • Thành viên
  • 229 Bài viết

16,Chứng minh với mọi x,y,z dương ta có
$(\frac{x}{y}+\frac{z}{\sqrt[3]{xyz}})^2+(\frac{y}{z}+\frac{x}{\sqrt[3]{xyz}})^2+(\frac{z}{x}+\frac{y}{\sqrt[3]{xyz}})^2\geq 12$

ta có : $$(1+1+1)[(\frac{x}{y}+\frac{z}{\sqrt[3]{xyz}})^2+(\frac{y}{z}+\frac{x}{\sqrt[3]{xyz}})^2+(\frac{z}{x}+\frac{y}{\sqrt[3]{xyz}})^2]\geq (\frac{x}{y}+\frac{z}{\sqrt[3]{xyz}}+ \frac{y}{z}+\frac{x}{\sqrt[3]{xyz}}+\frac{z}{x}+\frac{y}{\sqrt[3]{xyz}})^2\geq 36$$.
bất đt đầu dùng S-V , bất đt sau dùng AM-GM :D
từ đó suy ra đpcm

Bài viết đã được chỉnh sửa nội dung bởi duong vi tuan: 01-01-2013 - 12:49

NGU
Hình đã gửi

#132
maitienluat

maitienluat

    Trung sĩ

  • Thành viên
  • 182 Bài viết

16,Chứng minh với mọi x,y,z dương ta có
$(\frac{x}{y}+\frac{z}{\sqrt[3]{xyz}})^2+(\frac{y}{z}+\frac{x}{\sqrt[3]{xyz}})^2+(\frac{z}{x}+\frac{y}{\sqrt[3]{xyz}})^2\geq 12$

Đặt $a^{6}=x,b^{6}=y,c^{6}=z$ (thật ra là để viết cho gọn)
Theo AM-GM: $\left ( \frac{a^{6}}{b^{6}}+\frac{c^{4}}{a^{2}b^{2}} \right )^{2}+\left ( \frac{b^{6}}{c^{6}}+\frac{a^{4}}{b^{2}c^{2}} \right )^{2}+\left ( \frac{c^{6}}{a^{6}}+\frac{b^{4}}{c^{2}a^{2}} \right )^{2}\geq 4\frac{a^{2}c^{2}}{b^{4}}+4\frac{b^{2}a^{2}}{c^{4}}+4\frac{c^{2}b^{2}}{a^{4}}\geq 12$
Đẳng thức xảy ra khi $x=y=z$

Bài viết đã được chỉnh sửa nội dung bởi maitienluat: 01-01-2013 - 16:09


#133
no matter what

no matter what

    Why not me

  • Thành viên
  • 397 Bài viết
Thôi thì xấu tốt gì cũng xin trình bày thêm 1 cách nữa cho bài 16
$(\frac{x}{y}+\frac{z}{\sqrt[3]{xyz}})^2\geq (\frac{x}{y}+\frac{3z}{x+y+z})^2\geq (2\sqrt{\frac{3xz}{y(x+y+z)}})^2=12.\frac{xz}{y(x+y+z)}$
Xây dựng các BĐT còn lại là xong :icon6:

Bài :19,Chứng minh với mọi a,b,c dương ta có
$\frac{a^2}{a^2+bc}+\frac{b^2}{b^2+ca}+\frac{c^2}{c^2+ab}\leq \frac{a+b+c}{2\sqrt[3]{abc}}$

#134
Tienanh tx

Tienanh tx

    $\Omega \textbf{Bùi Tiến Anh} \Omega$

  • Thành viên
  • 360 Bài viết
TOPIC vắng qá, cho 1 bài làm khuấy động phong trào nhé :D
Bài Toán: Cho $a+x=b+y=c+z=k$
Chứng minh rằng $ax+by+cz \leq k^2$ :ukliam2: :ukliam2: :ukliam2: :ukliam2:

Bài viết đã được chỉnh sửa nội dung bởi tienanh1999bp: 07-01-2013 - 20:41

$\cdot$ $( - 1) = {( - 1)^5} = {( - 1)^{2.\frac{5}{2}}} = {\left[ {{{( - 1)}^2}} \right]^{\frac{5}{2}}} = {1^{\frac{5}{2}}} =\sqrt{1}= 1$

$\cdot$ $\dfrac{0}{0}=\dfrac{100-100}{100-100}=\dfrac{10.10-10.10}{10.10-10.10}=\dfrac{10^2-10^2}{10(10-10)}=\dfrac{(10-10)(10+10)}{10(10-10)}=\dfrac{20}{10}=2$

$\cdot$ $\pi\approx 2^{5^{0,4}}-0,6-\left(\frac{0,3^{9}}{7}\right)^{0,8^{0,1}}$

$\cdot$ $ - 2 = \sqrt[3]{{ - 8}} = {( - 8)^{\frac{1}{3}}} = {( - 8)^{\frac{2}{6}}} = {\left[ {{{( - 8)}^2}} \right]^{\frac{1}{6}}} = {64^{\frac{1}{6}}} = \sqrt[6]{{64}} = 2$

 

 

 

 


#135
BlackSelena

BlackSelena

    $\mathbb{Sayonara}$

  • Hiệp sỹ
  • 1549 Bài viết

TOPIC vắng qá, cho 1 bài làm khuấy động phong trào nhé :D
Bài Toán: Cho $a+x=b+y=c+z=k$
Chứng minh rằng $ax+by+cz \leq k^2$ :ukliam2: :ukliam2: :ukliam2: :ukliam2:

Có hơi lỏng k em ...
$ax \leq \dfrac{(a+x)^2}{4} = \dfrac{k^2}{4}$
Thiết các bđt tương tự và cộng lại thì $\sum ax \leq \dfrac{3k^2}{4}$
Còn đề như em nói thì theo anh có thể biến tấu lại là $a^2+x^2 = \dfrac{k}{3}$ khi đó áp dụng C-S là xong.

Bài viết đã được chỉnh sửa nội dung bởi BlackSelena: 07-01-2013 - 21:29


#136
duong vi tuan

duong vi tuan

    Thượng sĩ

  • Thành viên
  • 229 Bài viết

Thôi thì xấu tốt gì cũng xin trình bày thêm 1 cách nữa cho bài 16
$(\frac{x}{y}+\frac{z}{\sqrt[3]{xyz}})^2\geq (\frac{x}{y}+\frac{3z}{x+y+z})^2\geq (2\sqrt{\frac{3xz}{y(x+y+z)}})^2=12.\frac{xz}{y(x+y+z)}$
Xây dựng các BĐT còn lại là xong :icon6:

Bài :19,Chứng minh với mọi a,b,c dương ta có
$\frac{a^2}{a^2+bc}+\frac{b^2}{b^2+ca}+\frac{c^2}{c^2+ab}\leq \frac{a+b+c}{2\sqrt[3]{abc}}$

em si nghĩ bài này cũng gần 1 tuần rồi > Lấy hết can đảm cho số (1,1,2) vào =>......
Chủ top đâu , vào giải thích hộ em phát >:) >:) >:) >:)
NGU
Hình đã gửi

#137
no matter what

no matter what

    Why not me

  • Thành viên
  • 397 Bài viết

em si nghĩ bài này cũng gần 1 tuần rồi > Lấy hết can đảm cho số (1,1,2) vào =>......
Chủ top đâu , vào giải thích hộ em phát >:) >:) >:) >:)

Thay vào vẫn đúng mà bạn :icon6: VT= 1,46666........<VP=1.587401. :icon6:
Chủ topic cũng chịu :( dạo này ngu toán nặng nề,đành đẻ lại cho lớp tài năng THCS thôi :(
Bài 21 (tạm rời bài 19- mọi người cứ vực dậy bất cứ lúc nào ) :icon6:
21,Chứng minh với mọi a,b,c dương có tích bằng 8 ta có
$\frac{a^2}{\sqrt{(1+a^3)(1+b^3)}}+\frac{b^2}{\sqrt{(1+b^3)(1+c^3)}}+\frac{c^2}{\sqrt{(1+a^3)(1+c^3)} }\geq \frac{4}{3}$

#138
duong vi tuan

duong vi tuan

    Thượng sĩ

  • Thành viên
  • 229 Bài viết

Thay vào vẫn đúng mà bạn :icon6: VT= 1,46666........<VP=1.587401. :icon6:
Chủ topic cũng chịu :( dạo này ngu toán nặng nề,đành đẻ lại cho lớp tài năng THCS thôi :(
Bài 21 (tạm rời bài 19- mọi người cứ vực dậy bất cứ lúc nào ) :icon6:
21,Chứng minh với mọi a,b,c dương có tích bằng 8 ta có
$\frac{a^2}{\sqrt{(1+a^3)(1+b^3)}}+\frac{b^2}{\sqrt{(1+b^3)(1+c^3)}}+\frac{c^2}{\sqrt{(1+a^3)(1+c^3)} }\geq \frac{4}{3}$

Hic làm ko ra mình hoá rồ , Xin lỗi bạn...
Bài 21: áp dụng :$$a^3+1\leq (\frac{a^2+2}{2})^2$$ ta chứng minh:
$$\sum 4\frac{a^2}{(a^2+2)(b^2+2)}\geq \frac{4}{3}\Leftrightarrow 2(a^2+b^2+c^2)+(ab)^2+(cb)^2+(ac)^2\geq 72$$
bất đẳng thức cuối đúng theo AM_GM
NGU
Hình đã gửi

#139
duong vi tuan

duong vi tuan

    Thượng sĩ

  • Thành viên
  • 229 Bài viết
Bài 22:)$3\sqrt[9]{\frac{9a(a+b)}{2(a+b+c)^2}}+\sqrt[3]{\frac{6bc}{(a+b)(a+b+c)}}\leq 4$
ta có:$\sqrt[3]{\frac{6bc}{(a+b)(a+b+c)}}\leq \frac{1}{3}(\frac{2b}{a+b}+\frac{3c}{a+b+c}+1)$
và:$3\sqrt[9]{\frac{2a}{a+b}.\frac{3}{2}\frac{a+b}{(a+b+c)}.\frac{3}{2}\frac{a+b}{(a+b+c)}}\leq \frac{3}{9}(\frac{2a}{a+b}+\frac{3}{2}\frac{a+b}{(a+b+c)}+\frac{3}{2}\frac{a+b}{(a+b+c)}+6)$
từ 2 cái trên ta suy ra đpcm
NGU
Hình đã gửi

#140
duong vi tuan

duong vi tuan

    Thượng sĩ

  • Thành viên
  • 229 Bài viết
23,Chứng mi9nh với mọi a,b,c dương có tổng bình phương bằng 1
$a+b+c+\frac{1}{abc}\geq 4\sqrt{3}$. Tổng bình phương =1 ??????
giả sử a là min(a,b,c)
do b,c<1 do đó ta đánh giá : $a\leq \frac{1}{9}=>\frac{1}{abc}\geq \frac{1}{a}\geq 9>4\sqrt{3}$
ko biết đúng ko.
NGU
Hình đã gửi




1 người đang xem chủ đề

0 thành viên, 1 khách, 0 thành viên ẩn danh